Developing a strategy to win a game of picking elements from $S_n$












1












$begingroup$



Given a integer $n>1$, Let $S_n$ be the group of permutations of the numbers $1,2,dots n$. Two players, $A$ and $B$, play the following game. Taking turns, they select elements(one element at a time) from the group $S_n$. It is forbidden to select an element that had been already selected. The game ends when the selected elements generate the whole group $S_n$. The player who made the last move loses. The first move is made by $A$. Which player has a winning strategy?




My attempt involves finding




What elements of $S_n$ can generate $S_n$?




I know that $(123 dots n)$ and $(12)$ can generate $S_n$.



But we are supposed to look for all other such set of elements which can generate $S_n$.



How do I solve this?










share|cite|improve this question











$endgroup$












  • $begingroup$
    "all other" sets might be a bit ambitious.
    $endgroup$
    – Lord Shark the Unknown
    Dec 23 '18 at 4:56










  • $begingroup$
    The probability that two random elements of $S_n$ generate $S_n$ approaches $1$ as $n to infty$.
    $endgroup$
    – Derek Holt
    Dec 23 '18 at 10:41










  • $begingroup$
    Please ask one question at a time.
    $endgroup$
    – Shaun
    Dec 23 '18 at 16:11










  • $begingroup$
    @shaun The actual question is "Second Question". I thought that the first question is necessary to solve the actual question. My attempt involves solving first question to solve second question
    $endgroup$
    – Rakesh Bhatt
    Dec 23 '18 at 17:04








  • 1




    $begingroup$
    Okay. I suggest that you edit the question, then, to make that clear. It might be closed as too broad otherwise, since it's easy to overlook such detail.
    $endgroup$
    – Shaun
    Dec 23 '18 at 17:06
















1












$begingroup$



Given a integer $n>1$, Let $S_n$ be the group of permutations of the numbers $1,2,dots n$. Two players, $A$ and $B$, play the following game. Taking turns, they select elements(one element at a time) from the group $S_n$. It is forbidden to select an element that had been already selected. The game ends when the selected elements generate the whole group $S_n$. The player who made the last move loses. The first move is made by $A$. Which player has a winning strategy?




My attempt involves finding




What elements of $S_n$ can generate $S_n$?




I know that $(123 dots n)$ and $(12)$ can generate $S_n$.



But we are supposed to look for all other such set of elements which can generate $S_n$.



How do I solve this?










share|cite|improve this question











$endgroup$












  • $begingroup$
    "all other" sets might be a bit ambitious.
    $endgroup$
    – Lord Shark the Unknown
    Dec 23 '18 at 4:56










  • $begingroup$
    The probability that two random elements of $S_n$ generate $S_n$ approaches $1$ as $n to infty$.
    $endgroup$
    – Derek Holt
    Dec 23 '18 at 10:41










  • $begingroup$
    Please ask one question at a time.
    $endgroup$
    – Shaun
    Dec 23 '18 at 16:11










  • $begingroup$
    @shaun The actual question is "Second Question". I thought that the first question is necessary to solve the actual question. My attempt involves solving first question to solve second question
    $endgroup$
    – Rakesh Bhatt
    Dec 23 '18 at 17:04








  • 1




    $begingroup$
    Okay. I suggest that you edit the question, then, to make that clear. It might be closed as too broad otherwise, since it's easy to overlook such detail.
    $endgroup$
    – Shaun
    Dec 23 '18 at 17:06














1












1








1


2



$begingroup$



Given a integer $n>1$, Let $S_n$ be the group of permutations of the numbers $1,2,dots n$. Two players, $A$ and $B$, play the following game. Taking turns, they select elements(one element at a time) from the group $S_n$. It is forbidden to select an element that had been already selected. The game ends when the selected elements generate the whole group $S_n$. The player who made the last move loses. The first move is made by $A$. Which player has a winning strategy?




My attempt involves finding




What elements of $S_n$ can generate $S_n$?




I know that $(123 dots n)$ and $(12)$ can generate $S_n$.



But we are supposed to look for all other such set of elements which can generate $S_n$.



How do I solve this?










share|cite|improve this question











$endgroup$





Given a integer $n>1$, Let $S_n$ be the group of permutations of the numbers $1,2,dots n$. Two players, $A$ and $B$, play the following game. Taking turns, they select elements(one element at a time) from the group $S_n$. It is forbidden to select an element that had been already selected. The game ends when the selected elements generate the whole group $S_n$. The player who made the last move loses. The first move is made by $A$. Which player has a winning strategy?




My attempt involves finding




What elements of $S_n$ can generate $S_n$?




I know that $(123 dots n)$ and $(12)$ can generate $S_n$.



But we are supposed to look for all other such set of elements which can generate $S_n$.



How do I solve this?







group-theory discrete-mathematics combinatorial-game-theory






share|cite|improve this question















share|cite|improve this question













share|cite|improve this question




share|cite|improve this question








edited Dec 23 '18 at 17:09







Rakesh Bhatt

















asked Dec 23 '18 at 4:40









Rakesh BhattRakesh Bhatt

967214




967214












  • $begingroup$
    "all other" sets might be a bit ambitious.
    $endgroup$
    – Lord Shark the Unknown
    Dec 23 '18 at 4:56










  • $begingroup$
    The probability that two random elements of $S_n$ generate $S_n$ approaches $1$ as $n to infty$.
    $endgroup$
    – Derek Holt
    Dec 23 '18 at 10:41










  • $begingroup$
    Please ask one question at a time.
    $endgroup$
    – Shaun
    Dec 23 '18 at 16:11










  • $begingroup$
    @shaun The actual question is "Second Question". I thought that the first question is necessary to solve the actual question. My attempt involves solving first question to solve second question
    $endgroup$
    – Rakesh Bhatt
    Dec 23 '18 at 17:04








  • 1




    $begingroup$
    Okay. I suggest that you edit the question, then, to make that clear. It might be closed as too broad otherwise, since it's easy to overlook such detail.
    $endgroup$
    – Shaun
    Dec 23 '18 at 17:06


















  • $begingroup$
    "all other" sets might be a bit ambitious.
    $endgroup$
    – Lord Shark the Unknown
    Dec 23 '18 at 4:56










  • $begingroup$
    The probability that two random elements of $S_n$ generate $S_n$ approaches $1$ as $n to infty$.
    $endgroup$
    – Derek Holt
    Dec 23 '18 at 10:41










  • $begingroup$
    Please ask one question at a time.
    $endgroup$
    – Shaun
    Dec 23 '18 at 16:11










  • $begingroup$
    @shaun The actual question is "Second Question". I thought that the first question is necessary to solve the actual question. My attempt involves solving first question to solve second question
    $endgroup$
    – Rakesh Bhatt
    Dec 23 '18 at 17:04








  • 1




    $begingroup$
    Okay. I suggest that you edit the question, then, to make that clear. It might be closed as too broad otherwise, since it's easy to overlook such detail.
    $endgroup$
    – Shaun
    Dec 23 '18 at 17:06
















$begingroup$
"all other" sets might be a bit ambitious.
$endgroup$
– Lord Shark the Unknown
Dec 23 '18 at 4:56




$begingroup$
"all other" sets might be a bit ambitious.
$endgroup$
– Lord Shark the Unknown
Dec 23 '18 at 4:56












$begingroup$
The probability that two random elements of $S_n$ generate $S_n$ approaches $1$ as $n to infty$.
$endgroup$
– Derek Holt
Dec 23 '18 at 10:41




$begingroup$
The probability that two random elements of $S_n$ generate $S_n$ approaches $1$ as $n to infty$.
$endgroup$
– Derek Holt
Dec 23 '18 at 10:41












$begingroup$
Please ask one question at a time.
$endgroup$
– Shaun
Dec 23 '18 at 16:11




$begingroup$
Please ask one question at a time.
$endgroup$
– Shaun
Dec 23 '18 at 16:11












$begingroup$
@shaun The actual question is "Second Question". I thought that the first question is necessary to solve the actual question. My attempt involves solving first question to solve second question
$endgroup$
– Rakesh Bhatt
Dec 23 '18 at 17:04






$begingroup$
@shaun The actual question is "Second Question". I thought that the first question is necessary to solve the actual question. My attempt involves solving first question to solve second question
$endgroup$
– Rakesh Bhatt
Dec 23 '18 at 17:04






1




1




$begingroup$
Okay. I suggest that you edit the question, then, to make that clear. It might be closed as too broad otherwise, since it's easy to overlook such detail.
$endgroup$
– Shaun
Dec 23 '18 at 17:06




$begingroup$
Okay. I suggest that you edit the question, then, to make that clear. It might be closed as too broad otherwise, since it's easy to overlook such detail.
$endgroup$
– Shaun
Dec 23 '18 at 17:06










1 Answer
1






active

oldest

votes


















1












$begingroup$

I guess for $n=1$, A must choose the identity, which generates $S_n$, so B wins.



For $n=2$ A wins by choosing the identity, and for $n=3$ A wins by choosing a $3$-cycle, such as $(1,2,3)$.



For $n ge 4$, all maximal subgroups of $S_n$ have even order, and the subgroup generated by the elements chosen so far will eventually be maximal, so B wins.






share|cite|improve this answer











$endgroup$














    Your Answer








    StackExchange.ready(function() {
    var channelOptions = {
    tags: "".split(" "),
    id: "69"
    };
    initTagRenderer("".split(" "), "".split(" "), channelOptions);

    StackExchange.using("externalEditor", function() {
    // Have to fire editor after snippets, if snippets enabled
    if (StackExchange.settings.snippets.snippetsEnabled) {
    StackExchange.using("snippets", function() {
    createEditor();
    });
    }
    else {
    createEditor();
    }
    });

    function createEditor() {
    StackExchange.prepareEditor({
    heartbeatType: 'answer',
    autoActivateHeartbeat: false,
    convertImagesToLinks: true,
    noModals: true,
    showLowRepImageUploadWarning: true,
    reputationToPostImages: 10,
    bindNavPrevention: true,
    postfix: "",
    imageUploader: {
    brandingHtml: "Powered by u003ca class="icon-imgur-white" href="https://imgur.com/"u003eu003c/au003e",
    contentPolicyHtml: "User contributions licensed under u003ca href="https://creativecommons.org/licenses/by-sa/3.0/"u003ecc by-sa 3.0 with attribution requiredu003c/au003e u003ca href="https://stackoverflow.com/legal/content-policy"u003e(content policy)u003c/au003e",
    allowUrls: true
    },
    noCode: true, onDemand: true,
    discardSelector: ".discard-answer"
    ,immediatelyShowMarkdownHelp:true
    });


    }
    });














    draft saved

    draft discarded


















    StackExchange.ready(
    function () {
    StackExchange.openid.initPostLogin('.new-post-login', 'https%3a%2f%2fmath.stackexchange.com%2fquestions%2f3050061%2fdeveloping-a-strategy-to-win-a-game-of-picking-elements-from-s-n%23new-answer', 'question_page');
    }
    );

    Post as a guest















    Required, but never shown

























    1 Answer
    1






    active

    oldest

    votes








    1 Answer
    1






    active

    oldest

    votes









    active

    oldest

    votes






    active

    oldest

    votes









    1












    $begingroup$

    I guess for $n=1$, A must choose the identity, which generates $S_n$, so B wins.



    For $n=2$ A wins by choosing the identity, and for $n=3$ A wins by choosing a $3$-cycle, such as $(1,2,3)$.



    For $n ge 4$, all maximal subgroups of $S_n$ have even order, and the subgroup generated by the elements chosen so far will eventually be maximal, so B wins.






    share|cite|improve this answer











    $endgroup$


















      1












      $begingroup$

      I guess for $n=1$, A must choose the identity, which generates $S_n$, so B wins.



      For $n=2$ A wins by choosing the identity, and for $n=3$ A wins by choosing a $3$-cycle, such as $(1,2,3)$.



      For $n ge 4$, all maximal subgroups of $S_n$ have even order, and the subgroup generated by the elements chosen so far will eventually be maximal, so B wins.






      share|cite|improve this answer











      $endgroup$
















        1












        1








        1





        $begingroup$

        I guess for $n=1$, A must choose the identity, which generates $S_n$, so B wins.



        For $n=2$ A wins by choosing the identity, and for $n=3$ A wins by choosing a $3$-cycle, such as $(1,2,3)$.



        For $n ge 4$, all maximal subgroups of $S_n$ have even order, and the subgroup generated by the elements chosen so far will eventually be maximal, so B wins.






        share|cite|improve this answer











        $endgroup$



        I guess for $n=1$, A must choose the identity, which generates $S_n$, so B wins.



        For $n=2$ A wins by choosing the identity, and for $n=3$ A wins by choosing a $3$-cycle, such as $(1,2,3)$.



        For $n ge 4$, all maximal subgroups of $S_n$ have even order, and the subgroup generated by the elements chosen so far will eventually be maximal, so B wins.







        share|cite|improve this answer














        share|cite|improve this answer



        share|cite|improve this answer








        edited Dec 23 '18 at 18:18

























        answered Dec 23 '18 at 17:26









        Derek HoltDerek Holt

        54.7k53574




        54.7k53574






























            draft saved

            draft discarded




















































            Thanks for contributing an answer to Mathematics Stack Exchange!


            • Please be sure to answer the question. Provide details and share your research!

            But avoid



            • Asking for help, clarification, or responding to other answers.

            • Making statements based on opinion; back them up with references or personal experience.


            Use MathJax to format equations. MathJax reference.


            To learn more, see our tips on writing great answers.




            draft saved


            draft discarded














            StackExchange.ready(
            function () {
            StackExchange.openid.initPostLogin('.new-post-login', 'https%3a%2f%2fmath.stackexchange.com%2fquestions%2f3050061%2fdeveloping-a-strategy-to-win-a-game-of-picking-elements-from-s-n%23new-answer', 'question_page');
            }
            );

            Post as a guest















            Required, but never shown





















































            Required, but never shown














            Required, but never shown












            Required, but never shown







            Required, but never shown

































            Required, but never shown














            Required, but never shown












            Required, but never shown







            Required, but never shown







            Popular posts from this blog

            Plaza Victoria

            In PowerPoint, is there a keyboard shortcut for bulleted / numbered list?

            How to put 3 figures in Latex with 2 figures side by side and 1 below these side by side images but in...